Proving that a vector field is conservative

JD_PM
Messages
1,125
Reaction score
156

Homework Statement



Screenshot (236).png


Homework Equations



$$F = \nabla \phi$$

The Attempt at a Solution



Let's focus on determining why this vector field is conservative. The answer is the following:

Screenshot (238).png

Screenshot (239).png

[/B]
I get everything till it starts playing with the constant of integration once the straightforward differential equations have been solved.

May you explain how does it conclude that ##\phi (x, y, z)## is a potential for ##F##?

Thanks.
 

Attachments

  • Screenshot (235).png
    Screenshot (235).png
    1.6 KB · Views: 321
  • Screenshot (236).png
    Screenshot (236).png
    6 KB · Views: 672
  • Screenshot (237).png
    Screenshot (237).png
    2.6 KB · Views: 319
  • Screenshot (238).png
    Screenshot (238).png
    19.8 KB · Views: 914
  • Screenshot (239).png
    Screenshot (239).png
    4.4 KB · Views: 430
Physics news on Phys.org
You want to find ##\phi## by integrating ##F##. You start by integrating with respect to ##x##, which tells you about the function form of ##\phi## with respect to ##x##, up to a constant ##C_1##, which is a constant wrt ##x## but can be a function of ##y## and ##z##. You then find how ##C_1## changes with ##y## by integrating over ##y##, and so on.
 
  • Like
Likes JD_PM
DrClaude said:
You want to find ##\phi## by integrating ##F##. You start by integrating with respect to ##x##, which tells you about the function form of ##\phi## with respect to ##x##, up to a constant ##C_1##, which is a constant wrt ##x## but can be a function of ##y## and ##z##. You then find how ##C_1## changes with ##y## by integrating over ##y##, and so on.

But I do not understand why ##\frac{\partial \phi}{\partial y} = \frac{\partial C_1}{\partial y}##
 
At that point in the solution, what is ##\phi## equal to? What do you get when you differentiate it with respect to ##y##?
 
  • Like
Likes JD_PM
Question: A clock's minute hand has length 4 and its hour hand has length 3. What is the distance between the tips at the moment when it is increasing most rapidly?(Putnam Exam Question) Answer: Making assumption that both the hands moves at constant angular velocities, the answer is ## \sqrt{7} .## But don't you think this assumption is somewhat doubtful and wrong?

Similar threads

  • · Replies 2 ·
Replies
2
Views
1K
  • · Replies 2 ·
Replies
2
Views
2K
  • · Replies 3 ·
Replies
3
Views
2K
  • · Replies 8 ·
Replies
8
Views
2K
Replies
15
Views
4K
  • · Replies 5 ·
Replies
5
Views
2K
  • · Replies 3 ·
Replies
3
Views
2K
Replies
10
Views
3K
Replies
4
Views
2K
  • · Replies 3 ·
Replies
3
Views
1K